Sei sulla pagina 1di 15

-1Minoufiya University Faculty of Engineering Automatic Control 4th.

Year Mechanical Power Department Shibin El-Kawm

Sheet No. (1) Laplaces Transformation: 1- Find Laplace Transform of the following Functions: a) f(t)= 0.03te-2t. 0 -2t Ae 0 Asin(t) t< 0 tu 0 t< 0 tu 0

b) f(t)=

c) f(t)=

d) x(t)= t + cos(6t) e) y(t)= e-tsin([t). 2- Find Laplaces Transform of the following functions: a) f(t)=0.03[1- cos(2t)]. b) f(t)= e-0.4tcos(12t). c) f(t)=sin(5t+T/3). d) f(t)= 1 2 0 + 1 t e) f(t)= 4 0 t< 1 1e t 3 3e te 4 t>4 sin(3t) cos(2t).

3- Obtain the time function, then find Laplaces Transform of the following graphs: g(t) 1

0 -1

-a-

f(t) 1

1 -b-

4- Use the Partial fraction expansions to find the Inverse Laplaces Transform of the following:

-2Minoufiya University Faculty of Engineering a) F(s)= 20 . 2 + 6s + 25)(s + 1) (s 12(s + 1) . s(s + 2) 2 (s + 3) 100 . 2 + 25)(s + 2) (s Automatic Control 4th. Year Mechanical Power Department Shibin El-Kawm

b) F(s)=

c) F(s)=

5- Find the Inverse Laplaces Transform of the following functions: a) G(s)= 1 . (s + 2)(s + 3) 10 s(s 2 + 4)(s + 1) . b) G(s)= 1 . 2 (s + 4) (s + 1) 2(s + 1) . s(s 2 + s + 2) 1 . 2 + 6s + 40) s(s + 2)(s

c) G(s)=

d) G(s)=

e) G(s)=

5 . 2 (s + 2)(s + 4) s

f) G(s)=

6- Using Laplaces Transform, please, solve the following differential equations: a) x .. . + 3x + 6x = 0, with x . (t) t=0 b) 2x .. .. . + 7x + 3x = 0, with x . (t) t=0 c) f . -t + 6f + 4f = e , with f . (t) t=0 = 0 and f(t) = 3 and x(t) t=0 =0 .

= 0 and x(t)

t=0

=x

t=0

=0 .

d)

d 3m(t) d 2 m(t) dm(t) +4 +9 + 36m(t) = cos(5t), with zero initial conditions . 3 2 dt dt dt

Sheet No. (2):

-3Minoufiya University Faculty of Engineering Automatic Control 4th. Year Mechanical Power Department Shibin El-Kawm

BLOCK DIAGRAM REDUCTION AND SIGNAL-FLOW GRAPH 1- Reduce the following block diagrams to the minimum open-loop form applying block diagram reduction method and then apply Masons rule (from the beginning). (a) G4 + G3 +

5i +

E+

Y3 G1

Y2

H1

G2

Y1

5o

H2 (b) H2 R + + X1

G1

X2

G2 H1

X3

G3

+ X4 +

X5

G4 (c) R + H3 E

G1

Y1

G2

Y2

G3

Y3

H1 H2

G4

2- Find C(s)/R2(s) from the following block diagram applying both methods of solution: H3 R1 + Y1 R21 Y3 + + Y43 G3 H2 H1 C

G1

Y2

G2

Sheet No. (3) Mathematical Modeling of Physical Systems

-4Minoufiya University Faculty of Engineering Automatic Control 4th. Year Mechanical Power Department Shibin El-Kawm

1- Find the transfer function of the system shown into the figure, where the force, f(t), is the input and the displacement, y(t), of the mass is the output.

m y(t) f(t) f

2-

Write the differential equations for each of the mechanical systems, shown into fig.(a and b). Assume zero initial conditions. Also obtain their transformed equations. x2(t) x1(t) f K1 M M f(t) 1 2 (a) f1 K2 f2

xo(t) M K1 f1
1

x1(t) M K2
2

x2(t) f(t)

(b)

3- Show that the transfer function of the network, shown in fig.(a), is given by: E (s) o 1 = and that of fig.(b) is given by : E (s) i X X s 2 + ( X + X + X )s + 1 1 2 1 2 12 E (s) o 1 = ; E (s) i 2 + ( X + X )s + 1 X X s 1 2 1 2 X =R C , 1 1 R 1
1

where

=R C , 2 R 2 2
2

and X

12

=R C . 1 2

R C Isolating amplifier Volt Gain K=1 Network (b)

e
2

e
2

Network (a)

Using block diagram reduction method, determine the transfer function Q2(s)/Q1(s) for each of figures (a) and (b), assuming that a1=a2=Tr2.

4-

-5Automatic Control 4th. Year Mechanical Power Department h Shibin El-Kawm 1 a


2

Minoufiya University Faculty of Engineering q


1

R1 h
1

a h R1
2

h Figure(b)

Figure(a)

5- The system, shown below, represents an electrically excited vibrating table. The input is voltage, v ,
i

and the output is the movement, xo. Construct block diagram for each equation, you estimate, draw the whole block diagram, then find the overall transfer function. R F=K I
1

xo M

Amplifier K
4

I e
i

L K Potentiometer K
3 2

6- An industrial process requires that the temperature, U, in a tank be held constant. The complete system arrangement, as shown into the figure below, has a voltage, vr(t), as the input. The actual temperature in the tank is measured by means of a thermocouple immersed in the tank. The voltage produced into the thermocouple is amplified to produce a voltage, vb (t)=K.U(t). The actuating signal, ve(t), is amplified to a voltage, vb1(t), which energizes the solenoid. The solenoid has a resistance, R, and inductance, L, such that the force, controlling the valve position, x(t), is given by fs(t)=Ks.is(t), against a resisting spring, K1, damper, F, of equivalent mass, M. The valve controls the flow of hot steam, q(t) inside the heating coil according to the simplified relationship; q(t)=Kvx(t). The temperature of the tank is related to the steam flow rate, in Laplaces domain, with 5(s)=Kc.Q(s)/(s+a). a) Draw block diagrams representing each component of this system. b) Determine the transfer function of each block. c) Determine the system transfer function, 5(s)/Vr(s)

q(t) Steam Generator Valve x(t) TANK M R,L

vr(t)

Amp. ve(t) Gain vb1(t) Ka

is(t)

-6Minoufiya University Faculty of Engineering Automatic Control 4th. Year Mechanical Power Department Shibin El-Kawm

7- Consider the potential servomechanism, shown into next Figure. Assume that the input to the system is the reference shaft position, r(t), and the system output is the output shaft position, c(t). Kp is the potentiometer error detector constant, U(t) is the motor shaft angular displacement. Draw the block diagram of each component, then draw the whole system block idiagram and find the a =Constant system transfer function, C(s)/R(s). Rf A Amp. r(t) e(t) Gain e1(t) i Lf Tm M f K Kp N1 T 1 U,Jm,Fm B VA=Kp .r(t) VB=Kp .c(t) n=N1/N2 T m=Km.if U T N 1 = 2 = 1. U T N 2 2 1 N2 c(t)

J,F c(t)

Sheet No. (4) 1- a) How do you know that certain given system is linear (from control point of view)? b) A schematic diagram of a steel rolling process is shown into next Figure. The following relationships are assumed to be known: e(t)=r(t)-b(t), b(t)=K1c(t), c(t)=a(t- E); E=d/v=delay time = constant, a(t)=n.Um, and the value of the resisting torque, T1(t), is proportional to the error value, e(t). (i) Describe the system by a set of differential and difference equations. (ii) Draw the block diagram representing the given system, then find C(s)/R(s), using block diagram reduction method only.

-7Minoufiya University Faculty of Engineering + r(t) Automatic Control Ra + Amp. e(t) Gain e (t) a Ka 4th. Year Mechanical Power Department Shibin El-Kawm La Const. Field Current
M

b(t)

T m,Um

T 1,Jm,Fm(Viscous Friction) Gear box and linear actuator

n K 1

Const. Velocity c(t) a(t) v (cm/sec). d (cm)

2- a) Explain the main function of servomechanism in control systems. b) Discuss, briefly, the useful effects of introducing a feedback loop into any given system (3 only). c) Estimate the control ratio, C(s)/R(s), for the system shown below applying Massons Rule and signal-flow graph representation. Find also the ratio B(s)/E(s). C R +

E(s) G3

G1

Y2 G4

G2 G5 G7 G8 C(s) R(s) Y2 and C(s) E(s) + + for the control system: Y1

B1 + E1(s)

G6

B E2(s) 2 ++

3- Find, using a signal-flow graph, the transfer functions: Y1 + 10/s 1+K1s R1 + E(s)

C(s)

Y3 1+K2s 5/s Y43

4- For the control system, shown below: a) Derive all necessary equations relating the system variables. b) Draw a block diagram for each equation. c) Draw the complete block diagram of the system with Er(s) as main input and 5(s) as main output d) Draw the signal-flow graph for this system, then find the control relations: X(s) 5(s) . and E (s) E (s) r r HINTS: 1- The force of attraction on the solenoid and its back emf are given : Fc=Kcic and eb=KL dx/dt. 2- The voltage across the generator field, ef, is equal to Kxx.

-8Minoufiya University Faculty of Engineering Automatic Control 4th. Year Mechanical Power Department Shibin El-Kawm

3- When the voltage across the solenoid coil is equal to zero, the spring, Ks, is unstreched and x=0. 4- Ignore voltage drop due to both inductance and resistance of the generator. 5- Ignore voltage drop due to inductance of the motor.
.

6- The tachometer should be treated as an ordinary generator in the feedback path; et=KtU . Rm Rf Cost. Field ig current x Kx if No Load B1 G eg Lf ef em M Je,Be,[e,U Tm Kb Ks Kg Km 1:1 D.C. TachoMechanical Link Spring [ meter Rc Kt M Solenoid Gear er e eb c ic Lc et

Sheet No. (5) Time Domain Analysis 1- For the following open-loop transfer functions with unity negative feedback systems, evaluate the steady-state errors corresponding to different input signals; unit step, unit ramp, and parabolic functions: 10(1 + 4s) 500 a) G(s) = , b) G(s) = s(1 + 0.1s) (1 + 0.1s)(1 + 0.2s) c) G(s) = K(1 + 2s)(1 + 4s) , s 2 (s 2 + 2s + 10) d) G(s) = 50 (1 + 0.1s)(1 + 2s)

2- a) Define the maximum overshoot in a control system, draw suitable illustrative curve(s).

-9Minoufiya University Faculty of Engineering Automatic Control 4th. Year Mechanical Power Department Shibin El-Kawm

[2 C(s) n b) For a second-order system, represented by : = R(s) s 2 + 2^[ s + [ 2 n n percentage maximum overshoot if the system is subjected to a unit-step input.

, deduce the

c) A unity negative feedback control system is subjected to a unit-step input. The open-loop transfer K function of this system is given by: G(s) = . The gain, K, must be changed in order to s(1 + X s) decrease the percentage maximum overshoot from 75% to 25% . Compute the damping ratio, ^, in both cases, then evaluate the ratio between the values of gain in 1st. and 2nd. case assuming a constant value for X . [2 n s(s2 + 2^[ s + [ 2 ) n n

HINT: Laplaces inverse transform of -^[ t n sin [

is given by: 1  ^2 -^

1+

1 1  ^2

1  ^ 2 t - J where tan J =

3- It is known that the tachometric feedback path affects the relative damping of a system. What is the suitable value of Kf in order to keep the value of the damping ratio, ^, of the system, shown into the figure below, at 0.707 . + 15 R(s) C(s) + s(s + 2)

Kf .s

4- A servomechanism is represented by the following differential equation: d2U + 4.8 dU = 144 I ; where I is the actuating error signal = r - 0.5 U, r is the reference input, and U is dt dt2 the output shaft position. Find the value of damping ratio, ^, natural frequency, [ n, and the damped frequency, [ d.

5- The transient response of a mechanical vibratory system is shown below. For P=2 lb. force applied suddenly, find K, M, and D.

-10Minoufiya University Faculty of Engineering Automatic Control 4th. Year Mechanical Power Department Shibin El-Kawm

6- One objective of the paper-making process is to maintain uniform consistency of the stock output as it progresses to drying and rolling. A diagram of the thick stock consistency dilution control system is shown into the figure below with its corresponding signal-flow graph representation. The amount of water added determines the consistency. Let H(s)=1, G (s) = K , and G(s) = 1 . Determine: a) The closed-loop transfer function c 1 + 10s 1+ 2 s T(s)=C(s)/R(s). b) The sensitivity of T with respect to K. c) The steady-state error due to a step change in the desired consistency, R(s)=A/s. E(s) C(s) U(s) R(s) Gc(s) 1 G(s) Desired Consistency Water R(s) U(s) M(s)

Valve

Controller

-1 Consistency Measurement

H(s) M(s)

Mixing

To Paper Making

7- For the open-loop transfer function G(s)H(s) =

K(1 + Ts) : s(1 + 0.1s)(1 + 0.01 s) a) Find the values of T and K so that the damping ratio and natural frequency of the secondorder part of the closed-loop characteristic equation are of values 0.2 and 100 rad./sec. respectively. (i.e. by ignoring orders higher than the 2nd.). b) determine the steady-state error of this system due to each of the following inputs: (i) a unit step-input, (ii) a unit ramp-input.

8- a) Define the sensitivity of a control system to a parameter change. b) Two feedback control systems, shown below, are represented by signal-flow graphs as: R(s) K1 K2 C(s)

-0.09

-0.09

R(s) K1 Minoufiya University 1 Faculty of Engineering

K2

C(s)

-114th. Year Mechanical Power Department Shibin El-Kawm

Automatic Control

-0.0099
C(s) T = 1 R(s) (i) Evaluate the closed-loop transfer functions T 1(s) and T2(s). (ii) Show that T1(s)=T 2(s)=100 if K1=K2=100. (iii) Compute the sensitivity of each system to changes in K1 for nominal values of K1=K2=100.

9- Determine the system transfer function of Fig.(a), then evaluate the transfer function, Hx(s), such that the closed-loop transfer function of the system in Fig.(b) is equivalent to the closed-loop transfer function of the system in Fig.(a). Find the steady-state response of C to a unit step-input. Then compute the steady-state errors,; e(t)ss and ex(t)ss for each of unit step-input and unit rampinput for the system of Fig.(b) only. R(s) + E(s) 15000 G (s) = x s(s + 5)(1 + 0.01s) C(s)

Figure(a) 1 H(s) = 1 + 0.1s R(s) + Ex(s) + + Figure(b) Hx(s) E(s) C(s) Gx(s)

Sheet No. (6) 1- Determine the resonance peak, Mr, and resonant frequency, [ r, for the system: C(s) 5 = R(s) s 2 + 2s + 5

2- Sketch Polar Plot for each of the following transfer functions:

-12Minoufiya University Faculty of Engineering Automatic Control 4th. Year Mechanical Power Department Shibin El-Kawm

5 1 + 2s 10 b) G(s) = s(1 + 2s) a) G(s) = c) G(s) = 20 (1 + 3s)(1 + 5s)

d) G(s) =
e) G(s) =

30 s(1 + 3s)(1 + 5s)


10(1 + s) s(1 + 0.2s)(1 + 0.5s)

3- Construct the Phase plot (i.e. Phase angle vs. frequency) only for the following open-loop functions: 4 a) G(s)H(s) = s(1 + s)(1 + 0.2s)2 b) G(s)H(s) = c) G(s)H(s) = d) G(s)H(s) = 4(1 + 0.5s) s 2 (1 + 0.2s)(1 + 0.1s) 4(1 + 0.1s)(1 + 0.025s) s3 (1 + 0.005s)(1 + 0.001s) 3 2 + 2s + 3) s(s

4- Draw the Bode plots for the following open-loop transfer functions: 200 a) G(s) = s(1 + 0.5s)(1 + 0.1s) b) G(s) = c) G(s) = d) G(s) = e) G(s) = 0.2s2 (1 + 0.4s)(1 + 0.04s) 5(0.6s + 1) s 2 (4s + 1) 0.1s(1 + 0.2s) s(s2 + 16s + 100) 10(s + 3) s(2 + s)(s2 + s + 2)

Hint: for ^=0.8, errors -4 and -1.2 at corner frequency and at one octave below and above respectively. for ^=0.354, errors 3 and 1.7 at corner frequency and at one octave below and above respectively.

5- Estimate the transfer function of the following systems represented by the Magnitude plot, then draw the corresponding phase plot of each: (a) 20log M -20 db/dec 36 -40 db/dec 0 [1 8 4 -20 db/dec [3 [

-13Minoufiya University Faculty of Engineering Automatic Control 4th. Year Mechanical Power Department Shibin El-Kawm

20log M (b) -20 db/dec 18 -40 db/dec 0 0.33 -12 -18 [ [1


2

[3

-20 db/dec -40 db/dec

20log M -20 db/dec 40 -40 db/dec 20 db/dec 20log M

0 2.5

90 [ 0

0.5

1 2 40 db/dec [

(c)

-60 db/dec 60 db/dec (d)

6- For a system, represented by its open-loop transfer function G(s)H(s) =

4(s + 0.5) s(s + 0.2)(s + 1) gain margin, phase margin, phase crossover frequency, and gain crossover frequency.

, find the

7- Test the stability of the following control systems represented by their characteristic equations: (a) s4+6s3+21s2+36s+20=0 (b) 4s3+3s2+2s+5=0 (c) s4+3s2+4s+5=0 (d) s6+5s5+4s4+3s3+s2+4s - 4=0 (e) 3s4+10s3+5s2+5s+2=0

-14Minoufiya University Faculty of Engineering Automatic Control 4th. Year Mechanical Power Department Shibin El-Kawm

(f) s5+s4+2s3+2s2+3s+5=0 (g) s6+2s5+8s4+12s3+20s2+16s+16=0 8- Find the range of values for K in order to maintain a stable condition in each of the following system: (a) s3+34.5s2+7500s+7500K=0 (b) s3+3Ks2+(K+2)s+4=0 K (c) G(s) = , H(s) = 1 s(s + 2)(s 2 + 3) (d) s4+5s3+5s2+4s+K=0 with negative feedback.

9- A PD controller is used into a control system, shown below, in order to maintain a steady-state error less than 1 for all positive values of the controller constant, b, when the system is subjected to a 6 unit step-input signal. R(s) + E(s) K(s + b) + 10 s2(s + 3) C(s)

(i) What will be the range of values of controller constant, K, such that the system is also stable? (use Routh Stability Table). (ii) Check a numerical value of 2.5 for K, if it satisfies both conditions of part (i), then find the corresponding range of values for constant b.

a)

A1 =

0 T , b1 = 1 , and c1 = [1 0] 0 -1.2 0.4 1 T b) A2 = , b2 = -2 , and c2 = [1 1] 0.4 - 1.8 -5 -6


1
T

2-

For the plant A1, b1, c1 (of problem no, 1), find the feedback gain, k, so that u=k x places the closed-loop poles of the system at s= -5 and s= -6, assuming K=1 manually, then use MATLAB to check your answers.

-15Minoufiya University Faculty of Engineering Automatic Control 4th. Year Mechanical Power Department Shibin El-Kawm

3- A control system is described by A = use matrix methods to find: (a) G (s).


p

1 T -4.5 - 1 T -1.5 - 2 , b = 1 , c = [1 0], and k = [0.5 1] . For K=1,

(b) Y(s)/R(s) and kp . (c) G eq(s) and *(s). (d) Heq(s). What will be changed if K=10?

0 . 0 1 4- Given a state-variable representation x = x + 1 u , y = [1 1]x, and u = r - kTx -1 - 1


.5] and K=1; (a) Draw the block diagram representation for the plant and for the system. (b) Using matrix methods, find *(s), *k(s), Heq(s), G eq(s), and Y(s)/R(s).

where kT =[1

Potrebbero piacerti anche